JBL Cardiology

Pataasin ang iyong marka sa homework at exams ngayon gamit ang Quizwiz!

A 60-year-old woman presents with chest discomfort, confusion, and weakness. The patient's husband tells you that she vomited once before EMS arrival. The patient's BP is 70/40 mm Hg, her pulse is 45 beats/min and weak, and her respirations are 14 breaths/min and unlabored. Which of the following is the MOST likely cause of her hypotension? A) Bradycardia B) Hypovolemia C) Myocardial ischemia D) Respiratory compromise

A

An 88-year-old female experienced a syncopal episode. She is now conscious, but complains of dizziness and generalized weakness, and her pulse is 170 beats/min and irregular. The EMT should recognize that this patient's syncope was MOST likely caused by: A) an acute cardiac dysrhythmia. B) increased parasympathetic tone. C) widespread systemic vasodilation. D) decreased sympathetic tone.

A

When an error occurs while using the AED, it is MOST often the result of: A) battery failure secondary to operator error. B) malfunction of the microchip inside the AED. C) misinterpretation of the patient's cardiac rhythm. D) excess patient movement during the analyze phase.

A AEDs are highly sensitive and specific in recognizing shockable rhythms (eg, V-Fib, pulseless V-Tach); this means that they are highly reliable. It would be extremely rare for the AED to recommend a shock when one is not indicated or fail to recommend a shock when one is indicated. When an error does occur, it is usually the operator's fault. The most common error is not having a charged battery. To avoid this problem, many AEDs are equipped with an alarm that warns the operator if the battery is not fully charged. Some of the older AEDs, however, are not equipped with this feature. Therefore, it is important to check the AED daily, exercise the battery as often as the manufacturer recommends, and always have a spare, fully charged battery on hand.

A patient whose artificial pacemaker has failed would MOST likely experience: A) weakness and bradycardia. B) hypertension and a headache. C) irreversible ventricular fibrillation. D) dizziness and excessive tachycardia.

A An artificial pacemaker is implanted in a person whose own cardiac electrical conduction system cannot maintain a regular rhythm and rate. If a pacemaker stops working, as when the battery wears out or an internal lead becomes detached, the patient often experiences syncope, dizziness, or weakness because of a slow heart rate (bradycardia). The pulse rate is typically less than 60 beats/min because the heart is beating without the stimulus of the pacemaker and without regulation of its own electrical conduction system, which may be damaged. In these circumstances, the heart tends to assume a fixed slow rate that may not be fast enough to maintain adequate cardiac output. In some cases, the patient's heart rate may be so low that he or she becomes hypotensive

Because of the property of automaticity, cardiac muscle cells are able to: A) contract spontaneously without a stimulus from a nerve source. B) rest for up to 2 minutes before initiating another electrical impulse. C) survive for long periods of time if oxygen does not reach the heart. D) generate an electrical impulse from the same pacemaker every time.

A Automaticity refers to the ability of the cardiac cells to spontaneously generate an electrical impulse without being stimulated by an outside nerve source. When cardiac muscle cells contract, so does the muscle. Clearly, the heart cannot rest for 2 minutes before initiating another impulse; extended "rest" of the heart is called cardiac arrest. Automaticity does not enable the heart to survive for any length of time without oxygen. The property of automaticity is present in every cardiac cell; it does not dictate where the common pacemaker site is located.

An 88-year-old female experienced a syncopal episode. She is now conscious, but complains of dizziness and generalized weakness, and her pulse is 170 beats/min and irregular. The EMT should recognize that this patient's syncope was MOST likely caused by: A) an acute cardiac dysrhythmia. B) increased parasympathetic tone. C) widespread systemic vasodilation. D) decreased sympathetic tone.

A Because her heart is beating so fast, you should suspect that her syncope occurred because of a drop in her blood pressure secondary to reduced ventricular filling time; the faster the heart beats, the less time there is for the ventricles to fill in between contractions. A fast, irregular pulse indicates the presence of a cardiac dysrhythmia. The sympathetic nervous system is responsible for increasing the heart rate, whereas the parasympathetic nervous system is responsible for slowing it down. Therefore, if parasympathetic nervous system function was increased or sympathetic nervous system function was decreased, you would expect the patient to experience bradycardia, not tachycardia.

The EMT is treating a man with chest pain and has assisted him with his nitroglycerin. Which of the following should the EMT anticipate during reassessment of this patient? A) Decreased blood pressure B) Increased level of anxiety C) Increased oxygen saturation D) Burning sensation in the chest

A Because nitroglycerin is a vasodilator, you should expect that the patient's blood pressure will be lower when you reassess it. Some patients experience only a minor decrease in blood pressure; other patients may experience a more significant decrease (one of the many reasons to reassess your patient). Nitroglycerin typically does not increase anxiety; the fact that it relieves their chest pain, however, may actually decrease their anxiety. Nitroglycerin has no effect on oxygen saturation. Some patients may experience a burning sensation under their tongue; a burning sensation in the chest is not typical following nitroglycerin administration.

In addition to chest pain or discomfort, a patient experiencing an acute coronary syndrome would MOST likely present with: A) ashen skin color, diaphoresis, and anxiety. B) severe projectile vomiting and flushed skin. C) irregular breathing and low blood pressure. D) profound cyanosis, dry skin, and a headache.

A Chest pain, pressure, or discomfort is the most common symptom of acute coronary syndrome (ACS; eg, unstable angina, acute myocardial infarction); it occurs in approximately 80% of cases. Patients with ACS are usually anxious and may have a feeling of impending doom. Nausea and vomiting are common complaints; however, projectile vomiting, which is typically associated with increased intracranial pressure, is uncommon. The skin is often ashen gray and clammy (diaphoretic) because of poor cardiac output and decreased perfusion. Less commonly, the patient's skin is cyanotic. Respirations are usually unlabored unless the patient has congestive heart failure, in which case respirations are rapid and labored; irregular breathing, however, is not common. Blood pressure may fall as a result of decreased cardiac output; however, most patients will have a normal or elevated blood pressure. If the patient complains of a headache, it is usually a side effect of the nitroglycerin he or she took before your arrival; ACS itself usually does not cause a headache.

The coronary arteries fill with blood when the: A) aortic valve closes. B) left ventricle contracts. C) aortic valve opens. D) right ventricle contracts.

A During systole (cardiac contraction), the aortic valve opens and covers the openings to the coronary arteries. During diastole (cardiac relaxation), the aortic valve closes; this causes blood to flow back down the aorta and fill the coronary arteries. Therefore, coronary perfusion occurs when the heart relaxes.

Aspirin is beneficial to patients experiencing an acute coronary syndrome because it: A) prevents a clot from getting larger. B) effectively relieves their chest pain. C) decreases cardiac workload by lowering the BP. D) destroys the clot that is blocking a coronary artery.

A Early administration of baby aspirin (160 to 325 mg) to patients with acute coronary syndrome (ACS) has clearly been shown to reduce mortality and morbidity. Aspirin (acetylsalicylic acid [ASA]) prevents the clot in a coronary artery from getting larger by inhibiting platelet aggregation; in other words, it makes the platelets less sticky, which means that they will be less likely to clump together. Aspirin does not relieve the chest pain or discomfort associated with ACS, nor does it reduce blood pressure. Furthermore, aspirin does not remove the clot that is blocking a coronary artery; cardiac catheterization or fibrinolytic (clot-buster) drugs are required to do this.

Following return of spontaneous circulation, the patient remains apneic. The EMT should: A) ventilate at 12 breaths/min and maintain an oxygen saturation of 95% to 99%. B) ventilate at 12 to 15 breaths/min and maintain an oxygen saturation of 100%. C) elevate the patient's lower extremities and ventilate at a rate of 20 breaths/min. D) remove the AED pads and ventilate the patient at a rate of 24 breaths/min.

A Following return of spontaneous circulation (ROSC) in a patient who remains apneic, you should ventilate at a rate of 10 breaths/min and maintain an oxygen saturation of 92% to 98%. An oxygen saturation of 100% should not be achieved; there is evidence that such a high oxygen saturation post-ROSC can have a negative impact on neurologic recovery. Keep the patient supine, leave the AED pads in place, and transport without delay.

A 60-year-old woman presents with chest discomfort, confusion, and weakness. The patient's husband tells you that she vomited once before EMS arrival. The patient's BP is 70/40 mm Hg, her pulse is 45 beats/min and weak, and her respirations are 14 breaths/min and unlabored. Which of the following is the MOST likely cause of her hypotension? A) Bradycardia B) Hypovolemia C) Myocardial ischemia D) Respiratory compromise

A Given the patient's presentation, her slow heart rate (bradycardia) is the most likely cause of her hypotension. When the heart rate is too slow or too fast, cardiac output can fall, resulting in hypotension. Hypovolemia would be expected to cause tachycardia, not bradycardia. There is no evidence of respiratory compromise in this patient; her breathing is unlabored and at a normal rate. Myocardial ischemia, which would explain her chest discomfort, may be caused by her bradycardia, but would not explain her blood pressure.

After delivering one shock with the AED and performing 2 minutes of CPR on a woman in cardiac arrest, you reanalyze her cardiac rhythm and receive a no shock advised message. This means that: A) she is not in a shockable rhythm. B) she has electrical activity but no pulse. C) her rhythm has deteriorated to asystole. D) the first shock restored a rhythm and pulse.

A If the AED gives a no shock advised message, it has determined that the patient is not in a shockable rhythm (eg, V-Fib, pulseless V-Tach). It does not indicate that the patient has a pulse, nor does it indicate that a normal cardiac rhythm has been restored. The AED does not distinguish pulseless electrical activity (PEA) from asystole; it only recognizes them as nonshockable. PEA is a condition in which organized cardiac electrical activity is present despite the absence of a pulse. Asystole is the absence of all cardiac electrical and mechanical activity. If the AED gives a no shock advised message, immediately resume CPR, starting with chest compressions, until ALS arrives or the patient starts to move.

After restoring a pulse in a cardiac arrest patient, you begin immediate transport. While en route to the hospital, the patient goes back into cardiac arrest. You should: A) advise your partner to stop the ambulance. B) begin CPR and proceed to the hospital. C) begin rescue breathing with a bag-mask device. D) analyze the patient's rhythm with the AED.

A If you restore a pulse in a cardiac arrest patient, the AED pads should remain attached to the patient's chest during transport in case cardiac arrest reoccurs en route to the hospital. You should, however, turn the AED off or disconnect the pads from the AED. If cardiac arrest reoccurs, you should immediately tell your partner to stop the ambulance and assist you as you begin CPR. Remember that the AED will not analyze the cardiac rhythm if the patient is moving. Once your partner is available to assist, you should analyze the cardiac rhythm, defibrillate if indicated, and immediately resume CPR. Contact medical control as soon as possible, but not before performing CPR and defibrillation.

Your partner has applied the AED to a cardiac arrest patient and has received a shock advised message. While the AED is charging, you should: A) continue chest compressions until your partner tells you to stand clear. B) perform rescue breathing only until the AED is charged and ready to shock. C) cease all contact with the patient until the AED has delivered the shock. D) retrieve the airway equipment and prepare to ventilate the patient.

A It is important to minimize interruptions in CPR, especially chest compressions, when at all possible. All contact with the patient must cease while the AED is analyzing. However, if the AED gives a shock advised message and begins charging, you should resume chest compressions until the AED is charged and ready to deliver the shock; at this point, you should cease contact with the patient. As soon as the AED delivers the shock, immediately resume CPR starting with chest compressions.

Shortly after assisting a 60-year-old woman with her second nitroglycerin treatment, she tells you that she is lightheaded and feels as if she is going to faint. Her symptoms are MOST likely due to: A) low blood pressure. B) an irregular heartbeat. C) nervousness and anxiety. D) a drop in her blood sugar.

A Nitroglycerin (NTG) is a vasodilator; as such, it may cause a drop in blood pressure (hypotension) in some patients. Signs and symptoms of hypotension include dizziness, lightheadedness, and fainting (syncope), among others. For this reason, you should always assess a patient's blood pressure before and after administering nitroglycerin. If the patient's systolic BP is less than 100 mm Hg, NTG should not be given. The patient is probably nervous and anxious, and may even have an irregular heartbeat; however, hypotension is a more likely cause of her symptoms. NTG does not affect a patient's blood sugar level.

By which of the following mechanisms does nitroglycerin relieve cardiac-related chest pain or discomfort? A) Vasodilation and increased myocardial oxygen supply B) Vasodilation and decreased myocardial oxygen supply C) Vasoconstriction and increased myocardial workload D) Vasoconstriction and increased cardiac oxygen demand

A Nitroglycerin is a vasodilator that causes smooth muscle relaxation. Smooth muscle is found within the walls of the blood vessels. Nitroglycerin causes vasodilation, including dilation of the coronary arteries, which in turn increases the flow of oxygenated blood to the heart and reduces myocardial workload. However, care must be taken when administering nitroglycerin to a patient. Because of its vasodilatory effects, nitroglycerin can cause hypotension; therefore, it should not be given to patients with a systolic blood pressure of less than 100 mm Hg.

At the end of ventricular relaxation, the left ventricle contains 110 mL of blood. This is referred to as the: A) preload. B) afterload. C) stroke volume. D) cardiac output.

A Preload is the amount of pressure on the ventricular wall at the end of ventricular relaxation (diastole) and is influenced by the volume of blood in the ventricle just before it contracts. Afterload refers to the resistance that the ventricles must contract against. A patient with hypertension, for example, would have an increased afterload due to systemic vasoconstriction; the smaller the arteries, the greater the resistance the heart must contract against. Stroke volume is the volume of blood ejected from the ventricles in a single beat. Cardiac output is the volume of blood pumped by the heart each minute; it is calculated by multiplying the stroke volume and heart rate.

Which of the following is an abnormal finding when using the Cincinnati Stroke Scale to assess a patient who presents with signs of a stroke? A) One arm drifts down compared with the other side. B) One of the pupils is dilated and does not react to light. C) The patient's face is symmetrical when he or she smiles. D) Both arms drift slowly and equally down to the patient's side.

A The Cincinnati Stroke Scale is used to assess patients suspected of experiencing a stroke. It consists of three tests: speech, facial droop, and arm drift. Abnormality in any one of these areas indicates a high probability of stroke. To test arm drift, ask the patient to hold both arms in front of his or her body, palms facing upward, with eyes closed and without moving. Over the next 10 seconds, observe the patient's arms. If one arm drifts down toward the ground, you know that side is weak; this is an abnormal finding. To test for facial droop, have the patient smile, showing his or her teeth. The face should be symmetrical (both sides of the face should move equally). If only one side of the face moves well, you know that something is wrong with the part of the brain that controls the facial muscles. You should assess the pupils of a patient with a suspected stroke; however, this is not a component of the Cincinnati Stroke Scale.

All of the following are components of the Cincinnati Stroke Scale, EXCEPT: A) pupil size. B) facial droop. C) speech pattern. D) arm movement.

A The Cincinnati Stroke Scale, which tests speech, facial droop, and arm drift, is a reliable tool that should be used during your assessment of a patient suspected of having had a stroke. To test speech, ask the patient to repeat a simple phrase; he or she should be able to repeat the phrase without slurred speech. If the patient cannot repeat the phrase, or repeats it with slurred speech, a stroke should be assumed. To test facial movement, ask the patient to smile and show his or her teeth; both sides of the face should move symmetrically (equally). If only one side of the face is moving well (facial droop), a stroke should be assumed. To test arm movement, ask the patient to hold both arms in front of his or her body, palms up, with eyes closed and without moving. Over the next 10 seconds, watch the patient's hands. If one arm drifts down toward the ground, you know that side is weak. You should assess pupillary size, equality, and reactivity in any patient with a neurologic disorder; however, this is not a part of the Cincinnati Stroke Scale.

The myocardium receives its blood supply from the coronary arteries that branch directly from the: A) aorta. B) vena cavae. C) left atrium. D) right ventricle.

A The aorta, which is the largest artery in the human body, originates immediately from the left ventricle. The coronary arteries branch directly off of the ascending aorta, thus allowing the myocardium to receive blood that has the highest concentration of oxygen. The vena cavae (superior and inferior) return oxygen-poor blood from the body back to the right atrium, where it is pumped to the right ventricle and then to the lungs. The left atrium receives freshly oxygenated blood from the lungs and then pumps it to the left ventricle, through the aorta, and to the body.

Which of the following clinical signs would indicate increased sympathetic nervous system activity? A) Tachycardia B) Hypotension C) Constricted pupils D) Slow respiratory rate

A The autonomic nervous system is comprised of the sympathetic and parasympathetic nervous systems. The sympathetic nervous system, also called the "fight or flight" system, is responsible for making adjustments that allow the body to compensate for increased metabolic demand (ie, fear, shock, fever, etc.). The sympathetic nervous system increases the heart rate, increases respiratory rate and depth, dilates blood vessels in the muscles, constricts blood vessels in the digestive system, and causes pupillary dilation. Therefore, increased sympathetic activity would be expected to cause hypertension, tachycardia, and tachypnea. By contrast, the parasympathetic nervous system, also called the "rest and digest" system, is responsible for slowing the respiratory and heart rates, constricting blood vessels in the muscles, dilating the blood vessels in the digestive system, and constricting the pupils.

A 58-year-old man is found unresponsive by his wife. According to his wife, he was complaining of a "dull ache" in his chest the day before, but refused to allow her to call 911. His blood pressure is 70/50 mm Hg, his pulse is 120 beats/min and weak, and his respirations are 28 breaths/min and labored. Further assessment reveals that his skin is cool, pale, and clammy. You should suspect: A) cardiogenic shock. B) hypovolemic shock. C) acute septic shock. D) pulmonary embolism.

A The patient in this scenario likely experienced an acute myocardial infarction the day before. However, because he refused medical care, the infarction has likely damaged a significant portion of his heart, resulting in cardiogenic shock. Cardiogenic shock occurs when the heart fails as a pump and can no longer meet the metabolic needs of the body; it has a very high mortality rate. The patient's labored breathing is likely caused by pulmonary edema, which occurs when blood backs up into the lungs because the heart cannot effectively pump it forward. There is no evidence of hypovolemia (ie, poor skin turgor) or sepsis (ie, fever). A pulmonary embolism typically presents with an acute onset of pleuritic (sharp) chest pain and difficulty breathing, and is also commonly associated with cyanosis.

Which of the following patients is the BEST candidate for fibrinolytic therapy? A) 50-year-old man who has slurred speech and left arm weakness that started 45 minutes ago; history of a heart attack 8 months ago B) 53-year-old woman who experienced a sudden, severe headache and is now unresponsive; history of a hemorrhagic stroke 3 years ago C) 59-year-old man whose mental status has declined over the past 12 hours; history of renal failure, hypertension, and type 2 diabetes D) 63-year-old woman had a syncopal episode and is now confused; she takes blood thinners and has a history of deep vein thrombosis

A To be the most effective, fibrinolytic medications for stroke should be administered within the first 3 hours following the onset of signs and symptoms. In some patients, this time frame may be extended to 4.5 hours. Any history of intracranial hemorrhage, regardless of how long ago it was, is a contraindication for fibrinolytic therapy. Other contraindications include bleeding disorders (ie, hemophilia), use of blood thinning medications (ie, Coumadin, Eliquis, Pradaxa), and any active hemorrhage. The ultimate decision to initiate fibrinolytic therapy is the physician at the emergency department, in consultation with the neurologist. However, EMS can, through a targeted assessment, identify those who would most likely benefit from it.

Which of the following statements regarding sudden cardiac arrest and ventricular fibrillation is correct? A) For each minute that defibrillation is delayed, the chance of survival decreases by as much as 10%. B) High-quality CPR often reverses ventricular fibrillation if it is initiated within 2 minutes of the onset. C) Most patients develop ventricular fibrillation within 10 minutes after the onset of sudden cardiac arrest. D) Patients with ventricular fibrillation are typically unconscious, are apneic, and have a weak and irregular pulse.

A Ventricular fibrillation (V-Fib), a chaotic quivering of the heart muscle, is the most common dysrhythmia that results in sudden cardiac arrest (SCA). In V-Fib, the heart is not beating effectively and is not pumping blood; therefore, the patient will not have a pulse. The single most important treatment for V-Fib is early defibrillation. Even if CPR is begun right at the time of the patient's collapse, the likelihood of survival decreases by as much as 10% for each minute that defibrillation is delayed. CPR alone rarely, if ever, converts V-Fib to a cardiac rhythm with a pulse. V-Fib is often a transient dysrhythmia, and the window of opportunity for successful defibrillation is very narrow. After 10 minutes of cardiac arrest, most patients deteriorate to asystole (absence of electrical and mechanical activity in the heart).

Which of the following questions would be MOST appropriate to ask when assessing a patient with chest pain? A) What does the pain feel like? B) Does the pain radiate to your arm? C) Would you describe the pain as sharp? D) Is the pain worse when you take a deep breath?

A When questioning any patient about any type of pain, you should avoid asking leading questions that can simply be answered yes or no. To obtain the most reliable assessment, open-ended questions should be asked to allow the patient to describe the quality of the pain in his or her own words.

Which of the following describes the MOST appropriate method of performing chest compressions on an adult patient in cardiac arrest? A) Compress the chest at least 2 inches, allow full recoil of the chest after each compression, minimize interruptions in chest compressions B) Allow full recoil of the chest after each compression, compress the chest to a depth of 2 inches, deliver compressions at a rate of at least 80/min C) Do not interrupt chest compressions for any reason, compress the chest no more than 1½ inches, allow partial recoil of the chest after each compression D) Minimize interruptions in chest compressions, provide 70% compression time and 30% relaxation time, deliver compressions at a rate of 100/min

A Which of the following describes the MOST appropriate method of performing chest compressions on an adult patient in cardiac arrest? A) Compress the chest at least 2 inches, allow full recoil of the chest after each compression, minimize interruptions in chest compressions B) Allow full recoil of the chest after each compression, compress the chest to a depth of 2 inches, deliver compressions at a rate of at least 80/min C) Do not interrupt chest compressions for any reason, compress the chest no more than 1½ inches, allow partial recoil of the chest after each compression D) Minimize interruptions in chest compressions, provide 70% compression time and 30% relaxation time, deliver compressions at a rate of 100/min

The energy setting for a biphasic AED: A) is typically 360 joules. B) is manufacturer specific. C) must be manually set by the EMT. D) increases by 50 joules with each shock.

B A biphasic AED sends energy in two directions simultaneously. This is advantageous because it produces a more efficient defibrillation than monophasic defibrillation, which sends energy in only one direction, and uses a lower energy setting. The energy setting on a biphasic AED is device specific, but typically ranges from 120 to 200 joules. AEDs are preprogrammed by the manufacturer to deliver a specific amount of energy, which eliminates the need to manually set the energy level. Some biphasic AEDs deliver the same amount of energy for each shock; others may escalate the energy level for each shock. Refer to the manufacturer's documentation that accompanies the AED you use regarding the amount of biphasic energy it delivers.

A middle-aged woman took three of her prescribed nitroglycerin tablets after she began experiencing chest pain. She complains of a bad headache and is still experiencing chest pain. You should assume that: A) her blood pressure is elevated. B) she has ongoing cardiac ischemia. C) her nitroglycerin is no longer potent. D) her chest pain is not cardiac related.

B A headache and/or a bitter taste under the tongue are common side effects of nitroglycerin (NTG) that many patients experience. If the patient does not experience these side effects, the NTG may have lost its potency. However, if a patient with chest pain takes NTG and experiences these side effects, but still has chest pain, you should assume that his or her pain is the result of cardiac ischemia, a relative deprivation of oxygen to the heart. NTG is a vasodilator drug; therefore, it would lower her blood pressure, not raise it. Any patient with nontraumatic chest pain or pressure should be assumed to be experiencing cardiac ischemia, especially if the pain or pressure is not relieved with NTG.

Which of the following types of stroke would MOST likely present with a sudden, severe headache? A) Ischemic B) Hemorrhagic C) Thrombotic D) Embolic

B A stroke occurs when blood flow to a portion of the brain is interrupted. There are two types of stroke, both of which are acute events: ischemic and hemorrhagic. Hemorrhagic stroke is caused by a ruptured artery in the brain, resulting in intracerebral bleeding, cerebral ischemia, and increased intracranial pressure. A classic feature of a hemorrhagic stroke is a sudden, severe headache (indicates cerebral artery rupture) that is followed by rapid deterioration of the patient's condition. By contrast, ischemic stroke (accounts for approximately 87% of all strokes) occurs when a clot occludes a cerebral artery. If the clot forms locally, it is referred to as a thrombotic stroke; if the clot travels to the brain from another part of the body, it is referred to as an embolic stroke. Acute ischemic stroke typically presents with a sudden onset of confusion, facial droop, slurred speech, and weakness to one side of the body. A headache is uncommon in patients with acute ischemic stroke.

A 69-year-old man is unresponsive. He has a weak carotid pulse at 40 beats/min and his skin is cool and pale. From this information, the EMT should conclude that: A) he is in ventricular fibrillation. B) his cardiac output is reduced. C) he has a blocked coronary artery. D) his systolic BP is at least 90 mm Hg.

B Although the patient's pulse is slow and weak, it is present; this rules out ventricular fibrillation (V-Fib) because V-Fib does not produce a pulse. Without a 12-lead ECG and other advanced diagnostic tools, there is no way of knowing if the patient has a blocked coronary artery. An unresponsive patient with severe bradycardia; a weak pulse; and pale, cool skin likely has a BP that is less than 90 mm Hg. Therefore, the EMT should conclude from this patient's presentation that his cardiac output is reduced.

While performing CPR on a patient who is wearing an external defibrillator vest, the device alarms and a voice states that a shock is about to be delivered. What should you do? A) Remove the battery from the device and apply your AED B) Cease patient contact until the shock has been delivered C) Continue CPR because the vest delivers a low energy shock D) Allow it to shock, but immediately check for a carotid pulse

B An alternative to the implantable cardiac defibrillator is the external defibrillator vest. This device is a vest with built-in monitoring electrodes and defibrillation pads; it is worn by the patient under his or her clothing. The monitor provides alerts and voice prompts when it recognizes a lethal dysrhythmia and before a shock is delivered. Unlike the implantable defibrillator, the defibrillator vest uses high-energy shocks similar to an AED; therefore, you should avoid contact with the patient until the shock has been delivered. Immediately after the shock has been delivered, you should immediately resume chest compressions; do not stop to check for a pulse. Removing the battery or otherwise disabling the device when it is about to deliver a shock is clearly inappropriate. The vest should be left in place while CPR is being performed unless it interferes with chest compressions. If it is necessary to remove the vest, simply remove the battery from the monitor, remove the vest, and apply your AED.

When treating a patient with chest pain, pressure, or discomfort, you should FIRST: A) administer supplemental oxygen. B) place the patient in a position of comfort. C) request an ALS ambulance response to the scene. D) assess the blood pressure and give nitroglycerin.

B An important aspect of treating a patient with chest pain, pressure, or discomfort is to ensure that the patient is in a comfortable position. Most of the time, the patient will already be in this position upon your arrival. A comfortable position will help minimize anxiety, which in turn decreases cardiac oxygen consumption and demand. The decision to administer supplemental oxygen depends on the patient's oxygen saturation and/or whether or not respiratory distress is present. Following your assessment, if you feel that ALS support is needed, you should request it. If the patient has prescribed, unexpired nitroglycerin; the systolic blood pressure is greater than 100 mm Hg; and the patient has not taken the maximum of three doses, you should contact medical control to obtain permission to assist the patient in taking the nitroglycerin.

The pain associated with acute aortic dissection: A) typically comes on gradually and progressively worsens. B) is typically described as a stabbing or tearing sensation. C) is usually preceded by nausea, sweating, and weakness. D) originates in the epigastrium and radiates down both legs.

B Aortic dissection occurs when the inner layers of the aorta become separated, allowing blood to flow between the layers at high pressure. This separation of layers significantly weakens the aortic wall, making it prone to rupture. Signs and symptoms of acute aortic dissection include a sudden onset of a ripping, tearing, or stabbing pain in the anterior part of the chest or in the back between the scapulae. It may be difficult to differentiate the pain of acute aortic dissection from that of an acute myocardial infarction (AMI), but a number of distinctive features may help. The pain from an AMI is often preceded by other signs and symptoms (ie, nausea, indigestion, weakness, sweating [diaphoresis]). It tends to come on gradually and becomes more severe as time progresses, and is usually described as a crushing pain or as a feeling of heaviness or pressure. By contrast, the pain associated with aortic dissection is acute and is often of maximum intensity from the onset; it is typically described as a ripping, tearing, or stabbing sensation.

Which of the following is the MOST detrimental effect that tachycardia can have on a patient experiencing a cardiac problem? A) Increased blood pressure B) Increased oxygen demand C) Increased stress and anxiety D) Decreased cardiac functioning

B As the heart beats faster, it consumes and demands more oxygen. Tachycardia can be extremely detrimental to the patient with a compromised heart that is already deprived of oxygen. Keeping the patient calm cannot be overemphasized. The more stressed and anxious the patient gets, the faster his or her heart will beat; this is NOT a favorable effect in a patient experiencing acute cardiac ischemia!

A 65-year-old man has generalized weakness and chest pressure. He has a bottle of prescribed nitroglycerin, but states that he has not taken any of his medication. The EMT should: A) apply the AED and prepare the patient for immediate transport. B) administer up to 325 mg of aspirin if the patient is not allergic to it. C) assist the patient with his nitroglycerin with medical control approval. D) perform a secondary assessment and obtain baseline vital signs.

B Aspirin has clearly been shown to reduce mortality and morbidity associated with acute coronary syndrome (ACS) and should be given as early as possible; the dose is 160 to 325 mg. Even though this patient has chest pain and prescribed nitroglycerin, you must first complete a secondary assessment and obtain baseline vital signs. Medical control will need this information--specifically, the patient's blood pressure--to determine whether you should assist the patient with his nitroglycerin. The AED is not indicated for this patient because he is not in cardiac arrest.

In patients with heart disease, acute coronary syndrome is MOST often the result of: A) atrial damage. B) atherosclerosis. C) coronary artery spasm. D) coronary artery rupture.

B In most patients with acute coronary syndrome (ACS; eg, unstable angina, acute myocardial infarction), atherosclerosis is the underlying problem that causes heart disease. Atherosclerosis is a disorder in which calcium and a fatty material called cholesterol build up and form a plaque inside the walls of blood vessels, obstructing blood flow. ACS due to atherosclerosis usually occurs when a fragment of plaque ruptures and occludes a coronary artery; further occlusion occurs when platelets aggregate in the area and clump together. Less commonly, acute coronary artery spasm may result in ACS. The cause of acute coronary vasospasm is largely unknown. Rupture of a coronary artery is a rare cause of ACS. Atrial or ventricular damage is usually caused by, rather than the cause of, ACS

Ischemic heart disease is a condition in which: A) a portion of the heart muscle dies because of a prolonged lack of oxygen. B) there is a decrease in blood flow to one or more portions of the heart muscle. C) the coronary arteries dilate, thus preventing effective blood flow to the heart. D) an acute event leads to a significant decrease in the pumping force of the heart.

B Chest pain or discomfort that is related to the heart usually stems from a condition called ischemia (insufficient oxygen). Because of a partial or complete blockage of blood flow through one or more coronary arteries, the tissue of the heart muscle (myocardium) fails to get enough oxygen and nutrients relative to its needs. Therefore, ischemic heart disease is a condition involving a decrease in blood flow, and therefore oxygen, to one or more portions of the myocardium. If blood flow to the ischemic portion of the myocardium is not restored, it eventually dies (myocardial infarction). Dilation of the coronary arteries increases, not decreases, blood flow to the heart. If an event such as a myocardial infarction damages the heart and significantly decreases its ability to contract forcefully, heart failure may occur.

A 45-year-old woman calls EMS because of severe chest pain. When you arrive, she advises you that she has taken two of her husband's nitroglycerin tablets without relief. Her BP is 110/60 mm Hg, her pulse is 100 beats/min, her respirations are 16 breaths/min, and her oxygen saturation is 95%. You should: A) attach the AED, administer 100% oxygen, and contact medical control for advice. B) transport the patient without delay and monitor her blood pressure en route. C) Call medical control and request permission to assist the patient with one more NTG tablet. D) Apply oxygen, assess the patient's blood pressure, and give a third and final NTG tablet.

B Do not administer, or assist a patient with, a medication that is not prescribed to him or her. In this case, the best course of action is to transport the patient without delay and monitor her blood pressure en route. Additionally, if she has no drug allergies, you should administer up to 325 mg of chewable aspirin. Oxygen is not indicated for this patient; there is no evidence of respiratory compromise and her oxygen saturation is greater than 94%. The AED is applied only to patients who are in cardiac arrest; the patient in this scenario is not in cardiac arrest.

What occurs during recoil of the chest in between compressions? A) Positive pressure accumulates in the thoracic cavity B) A vacuum is created and blood returns to the heart C) Excessive amounts of air are expelled from the lungs D) Forward flow of blood moves to the body's vital organs

B During CPR, it is critical to allow the chest to fully recoil before providing the next compression. Recoil of the chest produces a vacuum, which pulls air into the lungs and draws more blood back to the heart. If only partial recoil occurs, positive pressure can accumulate in the chest, which would impair blood return to the heart. Forward blood flow and expulsion of air from the lungs occurs during the downstroke of the compression, not the recoil phase.

After defibrillating a man in cardiac arrest, you resume CPR. As you are about to reanalyze his cardiac rhythm 2 minutes later, your partner tells you she can definitely feel a strong carotid pulse. You should: A) continue with the rhythm analysis. B) assess the patient's breathing effort. C) ask her to obtain a blood pressure reading. D) remove the AED pads from the patient's chest.

B If return of spontaneous circulation (ROSC) occurs, your first action should be to reassess the patient's airway status and breathing effort. If the patient is still apneic or is breathing inadequately, continue ventilations and frequently reassess his or her pulse. If the patient is breathing adequately, administer oxygen as needed to maintain an oxygen saturation of greater than 94%. If the patient is still unresponsive, as is often the case, insert an appropriate airway adjunct (if not already done) to assist in maintaining airway patency. After reassessing airway and breathing, obtain the patient's blood pressure and treat hypotension if needed. Do not remove the AED pads from the patient's chest, even if ROSC has occurred. The risk of cardiac arrest is still high and the patient may need further defibrillation. You should, however, disconnect the pads from the AED or simply turn the AED off.

The AED analyzes your pulseless and apneic patient's cardiac rhythm and advises that a shock is NOT indicated. You should: A) assess for a pulse for no more than 10 seconds. B) resume CPR, starting with chest compressions. C) open the patient's airway and check for breathing. D) reanalyze the cardiac rhythm for positive confirmation.

B If the AED gives a no shock advised message, you should immediately resume CPR, starting with chest compressions. After 2 minutes of CPR, reanalyze the patient's cardiac rhythm and follow the AED voice prompts. You should not assess for a pulse if the AED gives a no shock message; this will simply cause an unnecessary delay in performing chest compressions. Rarely, if ever, does CPR alone restore a normal cardiac rhythm and pulse. If the AED electrodes are improperly applied, it will not analyze the patient's cardiac rhythm; instead, you will receive a "check patient" or "check electrodes" message. Continue CPR, rhythm analysis every 2 minutes, and defibrillation (if indicated) until ALS personnel arrive or the patient starts to move.

You are assessing a 70-year-old male who complains of pain in both of his legs. He is conscious and alert, has a blood pressure of 160/90 mm Hg, a pulse rate of 110 beats/min, and respirations of 14 breaths/min and unlabored. Further assessment reveals edema to both of his feet and legs and jugular venous distention. What should you suspect? A) Left heart failure B) Right heart failure C) Pulmonary edema D) Chronic hypertension

B If the right side of the heart is damaged, fluid collects in the body (edema), often showing in the feet and legs. The collection of fluid in the part of the body that is closest to the ground is called dependent edema. The swelling causes relatively few symptoms other than discomfort. Another feature of right heart failure is jugular venous distention, which is an indication of blood backing up into the systemic circulation. Left heart failure typically presents with shortness of breath due to fluid in the lungs (pulmonary edema), which indicates blood backing up from the left side of the heart into the lungs. In severe pulmonary edema, the patient may cough up pink, frothy sputum. Right heart failure and/or left heart failure are also referred to as congestive heart failure (CHF). Chronic hypertension cannot be established on the basis of a single blood pressure reading.

In which of the following patients is nitroglycerin contraindicated? A) 41-year-old male with crushing substernal chest pressure, a blood pressure of 160/90 mm Hg, and severe nausea B) 53-year-old male with chest discomfort, diaphoresis, a blood pressure of 146/66 mm Hg, and regular use of Levitra C) 58-year-old male with chest pain radiating to the left arm, a blood pressure of 130/64 mm Hg, and prescribed Tegretol D) 66-year-old female with chest pressure of 6 hours' duration, lightheadedness, and a blood pressure of 110/58 mm Hg

B Nitroglycerin is contraindicated in patients who do not have a prescription for nitroglycerin, in those with a systolic BP less than 100 mm Hg, and in patients who have taken medications for erectile dysfunction (ED) within the previous 24 to 48 hours. Such medications include sildenafil (Viagra), vardenafil (Levitra), and tadalafil (Cialis). Because bothED drugs and nitroglycerin cause vasodilation, concomitant use of these drugs may result in significant hypotension. Carbamazepine (Tegretol) is an anticonvulsant medication; there are no known interactions between Tegretol and nitroglycerin.

Which of the following patients is the BEST candidate for the administration of nitroglycerin? A) woman who has taken three doses of prescribed nitroglycerin without relief of chest pain B) A woman with chest pain, prescribed nitroglycerin, and a blood pressure of 104/76 mm Hg C) A man with chest pain, expired nitroglycerin spray, and a blood pressure of 110/80 mm Hg D) An elderly man with crushing substernal chest pain and a blood pressure of 80/60 mm Hg

B Nitroglycerin should be administered to patients who have the prescribed, unexpired drug with them and a systolic blood pressure of greater than 100 mm Hg. No more than three nitroglycerin tablets or sprays should be administered to a patient in the prehospital setting. An expired medication should never be administered to any patient, even if the medication is otherwise indicated for his or her condition.

Which of the following is the MOST appropriate response when a patient with chest pain asks you if he or she is having a heart attack? A) Yes, so I recommend going to the hospital. B) I don't know, but we will take good care of you. C) Probably not, but we should transport you to be safe. D) I believe you are, but only a physician can tell for sure.

B Patients experiencing chest pain often have a good idea about what is happening, so do not lie or offer false reassurance. Conversely, do not tell patients they are having a heart attack; EMTs are not trained to interpret the 12-lead ECG, and the 12-lead ECG is currently the only way a heart attack can be diagnosed in the prehospital setting. If asked, "Am I having a heart attack?", an appropriate response would be "I don't know for sure, but in case you are, we are going to take good care of you."

A 50-year-old man's implanted defibrillator has fired twice within the last hour. He is conscious and alert and complains of a "sore chest." Further assessment reveals that his chest pain is reproducible to palpation and is localized to the area of his implanted defibrillator. Treatment for him should include: A) application of the AED and transport to the hospital. B) prompt transport with continuous monitoring en route. C) deactivating his defibrillator by running a magnet over it. D) up to three doses of nitroglycerin and prompt transport.

B Patients who are at high risk for lethal cardiac dysrhythmias (ie, VF, VT) may have an automated implantable cardioverter/defibrillator (AICD). This small device is usually implanted in the upper left chest, just below the left clavicle. The AICD detects cardiac dysrhythmias and rapidly delivers a shock. When treating a patient whose AICD has fired, you should determine the number of times the device fired, assess vital signs, and transport to the hospital with continuous monitoring en route. Administer supplemental oxygen if the patient is experiencing respiratory distress and/or the oxygen saturation is less than 94%. Application of the AED is not indicated; however, if the patient develops cardiac arrest, you should use the AED as you normally would (remember to apply the pads at least 1 inch away from the implanted device). The pain that the patient is experiencing, which is reproducible and localized near his AICD, is likely musculoskeletal pain as the result of his AICD shocking him; therefore, nitroglycerin is not indicated. Because the AICD works so quickly (much faster than you can apply an AED), you should not make any attempt to deactivate it.

During your assessment of a 70-year-old man with crushing chest pain, you note that his blood pressure is 80/50 mm Hg. Your MOST important action should be to: A) give high-flow oxygen. B) transport without delay. C) keep the patient warm. D) assess his oxygen saturation.

B Patients with chest pain, pressure, or discomfort with a systolic BP less than 100 mm Hg should be transported to the hospital without delay. Hypotension in a patient with chest pain indicates cardiogenic shock due to severe cardiac damage and requires treatment that can only be given at the hospital. Any delay in transport delays definitive care and increases the patient's chance of death. High-flow oxygen, thermal management, and assessment of oxygen saturation are clearly important; however, your primary focus should be to get the patient to the hospital as soon as possible.

Prior to administering nitroglycerin to a patient with chest pain, you should: A) elevate the patient's lower extremities. B) obtain vital signs to detect hypotension. C) inquire about an allergy to salicylates. D) auscultate the patient's breath sounds

B Prior to assisting a patient with his or her prescribed nitroglycerin, there are two things that you must do: take the patient's vital signs and obtain authorization from medical control. Nitroglycerin is contraindicated for patients with a systolic blood pressure that is less than 100 mm Hg. If the patient develops hypotension after being given nitroglycerin, position him or her supine and transport without delay. Salicylates are a class of drugs that include aspirin, not nitroglycerin (nitroglycerin is a nitrate). Although you should inquire about medication allergies in general, it is not necessary to inquire specifically about an allergy to salicylates unless you are going to administer aspirin. Assessment of a patient with a possible cardiac or respiratory problem should include auscultation of breath sounds; however, this does not necessarily have to be done before assisting the patient with his or her nitroglycerin.

When applying the ECG electrodes, the negative (white) lead is placed on the: A) left arm. B) right arm. C) left leg. D) right leg.

B Proper lead placement is paramount in obtaining an accurate electrocardiographic tracing of the heart. Misplaced leads can produce erroneous ECG findings, or hide an abnormality that correct lead placement would otherwise have revealed. The negative (white) lead is placed on the right arm, the ground (black) lead is placed on the left arm, the positive (red) lead is placed on the left leg, and the green lead (completes the circuit) is placed on the right leg.

You should suspect that your patient has pulmonary edema if he or she: A) has swollen feet and ankles. B) cannot breathe while lying down. C) is hypertensive and tachycardic. D) has a dry, nonproductive cough.

B Pulmonary edema is often caused by failure of the left side of the heart. When the patient is lying down, he or she experiences worsened difficulty breathing (orthopnea) because more blood backs up in the lungs. Patients with severe pulmonary edema often produce pink, frothy sputum when they cough; this is another sign of blood backing up in the lungs. A dry, nonproductive cough is not common. Hypertension and tachycardia are common in patients with pulmonary edema; however, many other conditions can cause these findings. Swelling of the feet and ankles is commonly seen in patients with right heart failure, and occurs when blood backs up beyond the right atrium; it is not a common sign of left heart failure and pulmonary edema.

Switching compressors during two-rescuer CPR: A) should take no more than 15 seconds to accomplish. B) should occur every 2 minutes throughout the arrest. C) is necessary only if the compressor becomes fatigued. D) is performed after every 10 to 20 cycles of adult CPR.

B Rescuer fatigue leads to inadequate chest compression rate and/or depth. Fatigue is common after 1 minute of CPR, although the rescuer may not recognize it for 5 minutes or longer. Therefore, compressors should be changed every 2 minutes (after 5 cycles of CPR at a 30:2 ratio) throughout the resuscitation attempt. If the compressor is not switched until he or she recognizes the fatigue, the patient has likely been without effective chest compressions for at least 4 or 5 minutes. In general, interruptions in CPR should be infrequent and should not exceed 10 seconds. However, every effort should be made to switch compressors in less than 5 seconds

Which of the following clinical signs should alert the EMT that a patient has left heart failure? A) An irregular pulse B) Dyspnea on exertion C) Severe peripheral edema D) Jugular venous distention

B Signs of left heart failure are reflective of blood that backs up in the lungs and causes pulmonary edema. These include exertional dyspnea, paroxysmal nocturnal dyspnea, dyspnea while lying down (orthopnea), and coughing up blood (hemoptysis). Signs of right heart failure are reflective of blood that backs up in the systemic circulation. These include peripheral edema, jugular venous distention, and abdominal distention. An irregular pulse indicates the presence of a cardiac dysrhythmia, which could be present with left or right heart failure.

You assess an unresponsive 65-year-old man and find that he is apneic and pulseless. The patient's wife tells you that he has an automatic implanted cardioverter/defibrillator. After initiating CPR, you should: A) deactivate the implanted defibrillator by running a magnet over it. B) apply the AED as soon as possible and analyze his cardiac rhythm. C) ask the wife why and when he had the automatic defibrillator implanted. D) avoid using the AED because the implanted defibrillator is more effective.

B Some patients who are at high risk for sudden cardiac arrest due to ventricular fibrillation (V-Fib) have a small automatic implanted cardioverter/defibrillator (AICD). The AICD attaches directly to the heart and continuously monitors the cardiac rhythm, delivering a shock if V-Fib or another lethal dysrhythmia is detected. Regardless of whether the patient has an AICD, he or she should be treated like all other cardiac arrest patients. Perform CPR and use the AED as usual; however, you should ensure that the AED pads are at least 1 inch away from the implanted device. Generally, the electricity from the AICD is so low that it will have no effect on rescuers and therefore should not be of concern to you. Do not deactivate an implanted AICD, especially if it is working and delivering shocks as it is supposed to. When treating a cardiac arrest patient who has an AICD, your priority is to provide CPR and defibrillate with the AED if indicated, not to determine when and why the AICD was implanted.

When assessing a patient who has stroke-like symptoms, you should recall that: A) the majority strokes are caused by a ruptured cerebral artery. B) the patient may be unable to communicate, but can often understand. C) right-sided weakness indicates a stroke in the right cerebral hemisphere. D) fibrinolytic therapy must be given within 6 hours following the stroke.

B Some patients who have had a stroke may be unable to communicate (expressive aphasia), but they can often understand what is being said around them; be aware of this possibility. According to the American Heart Association, 87% of all strokes are caused by an occluded cerebral artery (ischemic stroke); strokes caused by a ruptured cerebral artery (hemorrhagic stroke) are less common. Because the left side of the brain controls the right side of the body, and vice versa, right-sided weakness (hemiparesis) indicates a stroke in the left cerebral hemisphere. Some patients who have had a stroke may benefit from fibrinolytic (clot-buster) therapy; however, to be most effective, this treatment must be given within the first 3 hours (4.5 hours in certain patients) following the onset of stroke symptoms.

A patient reports pain in the upper midabdominal area. This region of the abdomen is called the: A) peritoneum. B) epigastrium. C) mediastinum. D) retroperitoneum.

B The mid-upper region of the abdomen is referred to as the epigastrium because of its location over the stomach (epi = upon, gastric = stomach). This is a common site of pain or discomfort in patients experiencing a cardiac problem, which frequently causes them to attribute their pain or discomfort to indigestion.

A 57-year-old male presents with flu-like symptoms. He is conscious and alert; his skin is pink, warm, and dry; and he denies chest pain or respiratory distress. During your assessment, you cannot feel radial or carotid pulses, and are unable to obtain a blood pressure. Which of the following would MOST likely explain this? A) He is in the advanced stages of shock B) He has a left ventricular assist device C) He has a dissecting aortic aneurysm D) He recently had a coronary artery bypass

B The patient is clearly stable, as evidenced by his level of consciousness; normal skin, color, condition, and temperature; and absence of chest pain or respiratory distress. These findings rule out shock, especially advanced shock. Patients with acute aortic dissection typically present with severe, tearing chest pain that often radiates to the back; the patient has none of these symptoms. A coronary artery bypass graft (CABG) would not explain the absence of pulses and blood pressure. The only possible option is that he has a left ventricular assist device (LVAD) with a continuous flow pump. Patients with this type of LVAD do not have palpable pulses, despite the fact that their body is adequately perfused. An LVAD with a pulsatile pump, however, would generate a palpable pulse.

A 65-year-old woman complains of severe chest pain that radiates to her back, in between her shoulder blades. She tells you that this is the most severe pain that she has ever experienced, and that it has been intense since it began. Her past medical history includes hypertension. When you palpate her radial pulses, you note that they are unequal in strength. What should you suspect? A) Acute aortic dissection B) Unstable angina pectoris C) Acute hypertensive crisis D) Acute myocardial infarction

B The patient's clinical presentation is classic for acute dissection of the aorta. Dissection occurs when high pressure causes damage (dissection) through the layers of the artery until an aneurysm eventually forms. Hypertension is a major risk factor for aortic dissection. The patient with acute aortic dissection typically presents with a sudden onset of severe chest pain, which is often described as the worst pain they have ever felt. In many cases, the patient radiates to the back, in between the scapulae. In contrast to the pain associated with acute cardiac ischemia (ie, unstable angina, myocardial infarction), which often begins acutely and progressively worsens, the pain from aortic dissection is of maximal intensity from the onset. Depending on the degree of dissection, blood pressure changes (greater than 15 mm Hg) may be noted between arms, and pulses in the upper extremities may be unequal in strength. Since there is no blood pressure provided, and the patient's symptoms are not consistent with acute hypertensive crisis (ie, headache, photophobia, nausea, vomiting), there is no evidence that this is what is causing her symptoms.

A 72-year-old female complains of dyspnea that woke her from her sleep. Her feet and ankles are swollen, and auscultation of her lungs reveals crackles to both lung bases. She has a history of hypertension, type II diabetes, and a heart attack 2 years ago. Her BP is 170/94 mm Hg, her pulse is 110 beats/min and irregular, her respirations are 24 breaths/min and labored, and her oxygen saturation is 85% on room air. What should you suspect? A) Acute asthma attack B) Congestive heart failure C) Acute hypertensive crisis D) Emphysema exacerbation

B The patient's clinical presentation is consistent with congestive heart failure (CHF) with acute pulmonary edema. Dyspnea that awakens a person from sleep is called paroxysmal nocturnal dyspnea (PND) and is classic for CHF. Crackles auscultated over her lungs further support the diagnosis of pulmonary edema, and her low oxygen saturation indicates hypoxemia. Swelling to the feet and ankles suggests an element of right heart failure as well. Emphysema is unlikely because there is no mention of it in her medical history. Furthermore, patients with emphysema typically have dyspnea all the time, not just when they lie down to sleep. Acute asthma is also unlikely; one would expect wheezing (not crackles) in the lungs. Although the patient's BP is elevated, she has no other signs of acute hypertensive crisis, such as a headache, nausea and vomiting, and ringing in the ears

A 50-year-old man with a history of hypertension began experiencing chest pressure while mowing his lawn. After taking a baby aspirin and resting for approximately 10 minutes, the pressure in his chest completely resolved. Which of the following would BEST explain this? A) The patient's blood pressure increased after he ceased exertion, causing his chest pressure to resolve B) The cessation of exertion decreased the workload of the heart and blood flow around a coronary clot improved C) The aspirin dissolved a small clot in a coronary artery and reestablished blood flow to that part of the heart D) Blood flow to a part of the myocardium was restored because of constriction of a diseased coronary artery

B The patient's symptoms are consistent with angina. Angina is a condition in which cardiac oxygen demand exceeds the available supply; it indicates the presence of coronary artery disease. In this case, the patient's chest pressure resolved after he stopped exerting himself; this decreased the workload on the heart, provided for some dilation (not constriction) of the coronary arteries, and rebalanced cardiac oxygen supply and demand. As a result, blood flow through the diseased coronary artery improved and his chest pressure resolved. An increase in blood pressure, which increases cardiac workload, would not cause the patient's symptoms to resolve; it would likely make them worse. Aspirin will not dissolve a coronary thrombus; it will simply prevent platelets from clumping to the existing thrombus, so this would not explain why his symptoms resolved.

In most people, the inferior aspect of the left ventricle receives its blood supply from the: A) circumflex artery. B) right coronary artery. C) left main coronary artery. D) left anterior descending artery.

B The right coronary artery (RCA) supplies blood to the entire right side of the heart; in most people, it also supplies blood to the inferior aspect of the left ventricle in most people. The left main coronary artery branches into the left anterior descending (LAD) and circumflex arteries. The LAD artery supplies the ventricular septum and the anterior aspect of the left ventricle. The circumflex artery supplies the lateral and posterior aspects of the left ventricle.

The wall that separates the left and right sides of the heart is called the: A) carina. B) septum. C) pericardium. D) mediastinum.

B The septum is the wall that separates the left and right sides of the heart. There is a septum for both the atria and the ventricles. The carina is the bifurcation point of the trachea, and the mediastinum is the space between the lungs in which the heart, great vessels, and a portion of the esophagus lie. The pericardium is the sac that surrounds the heart and contains pericardial fluid.

A patient who is experiencing an acute myocardial infarction: A) most often describes his or her chest pain as being sharp or tearing. B) has chest pain or discomfort that does not change with each breath. C) often experiences relief of his or her chest pain after taking nitroglycerin. D) often complains of a different type of pain than a patient with angina.

B The type of chest pain or discomfort associated with acute myocardial infarction (AMI) is the same that is experienced by patients with angina pectoris (eg, dull, crushing, pressure, heaviness); thus, you cannot distinguish AMI from angina pectoris based solely on the type or quality of pain. Furthermore, the pain associated with AMI, like that of angina, often radiates to the arm, jaw, back, or epigastrium. Relative to other causes of chest pain or discomfort (eg, pleurisy, pneumothorax), the pain associated with AMI and angina does not worsen or improve when the patient takes a breath. Rest and nitroglycerin often relieve the pain associated with stable angina, but are less likely to relieve the pain associated with unstable angina or AMI.

Which of the following statements regarding ventricular fibrillation (V-Fib) is correct? A) Loss of consciousness occurs within minutes after the onset of V-Fib. B) In V-Fib, the heart is not pumping any blood and the patient is pulseless. C) Patients in V-Fib should be defibrillated after every 60 seconds of CPR. D) Any patient in V-Fib must receive CPR for 2 minutes prior to defibrillation.

B Ventricular fibrillation (V-Fib) is a disorganized, ineffective quivering of the heart muscle. No blood is pumped through the body and the patient is pulseless. Loss of consciousness occurs within seconds following the onset of V-Fib. Patients in V-Fib are treated with high-quality CPR and defibrillation every 2 minutes if needed. When treating a patient in cardiac arrest, whether the patient's arrest was witnessed or unwitnessed, begin immediate CPR and apply the AED as soon as it is available.

After attaching the AED and pushing the analyze button on an adult patient in cardiac arrest, the AED states that a shock is advised. Which cardiac rhythm is the patient MOST likely in? A) Asystole B) Ventricular fibrillation C) Ventricular tachycardia D) Pulseless electrical activity

B Ventricular fibrillation (V-Fib) is the most common initial cardiac dysrhythmia in adult cardiac arrest patients. V-Fib is a chaotic quivering of the heart muscle that does not produce a pulse and is due to a massive, uncontrolled electrical discharge of the cardiac cells. The most effective treatment for V-Fib is defibrillation. Some patients are in ventricular tachycardia (V-Tach) without a pulse, which is also treated with defibrillation. Although asystole and pulseless electrical activity (PEA) do not produce a pulse, they are not treated with defibrillation

A 44-year-old male complains of dizziness and lightheadedness. His BP is 88/62 mm Hg; his heart rate is 190 beats/min and weak; and his skin is cool, clammy, and pale. Which of the following would BEST explain why his BP is low? A) Widespread systemic vasoconstriction B) Decreased ventricular filling time C) Increased ventricular stroke volume D) Decreased myocardial oxygen demand

B When the heart is beating exceedingly fast, there is less time in between contractions for the ventricles to fill. As a result, stroke volume decreases and the blood pressure falls; this is the most likely cause of this patient's hypotension. Widespread vasoconstriction would increase the blood pressure, not decrease it. Decreased myocardial oxygen demand simply means that the heart is not working too hard; this does not affect blood pressure.

Common signs and symptoms of a hypertensive emergency include: A) unequal pupils, irregular pulse, and pallor. B) ringing in the ears, headache, and epistaxis. C) chest discomfort, weak pulses, and cool skin. D) vomiting without nausea and hemiparesis.

B Your patient is in shock, which is most likely the result of heart failure (cardiogenic shock). Immediate transport to the closest appropriate hospital is critical. Assess for and manage problems with airway, breathing, and circulation, and then get on the road! She needs definitive care that can be provided only at the hospital. Reassess her vital signs at least every 5 minutes, but do it en route! If you can rendezvous with an ALS unit en route, do so. However, you should not remain at the scene to wait for them. Nitroglycerin is clearly contraindicated for this patient; her BP is dangerously low.

You have analyzed a cardiac arrest patient's rhythm three times with the AED, separated by 2-minute cycles of CPR, and have received no shock messages each time. You should: A) consider terminating resuscitation. B) remove the AED and continue CPR. C) continue CPR and transport at once. D) request a paramedic unit at the scene.

C Although protocols vary from system to system, it is generally agreed that if you receive three consecutive no shock messages, separated by 2-minute cycles of CPR, you should continue CPR and transport at once; it is unlikely that the patient will convert to a shockable rhythm (eg, V-Fib, pulseless V-Tach). En route, coordinate a rendezvous with a paramedic unit if possible; waiting at the scene would simply delay further treatment. The decision to terminate resuscitative efforts is made by a physician, and in some cases, a paramedic, after adequately performed BLS and ALS have proved unsuccessful.

Which of the following is MOST indicative of a primary cardiac problem? A) Tachypnea B) Tachycardia C) Irregular pulse D) Sudden fainting

C An irregular pulse signifies an abnormality within the electrical conduction system of the heart. Tachycardia, sudden fainting (syncope), and tachypnea (rapid breathing) can indicate many things other than cardiac problems, such as shock, heat-related problems, and diabetic complications. You should always consider the possibility of a cardiac problem in a patient with an irregular pulse

With regard to the electrocardiogram (ECG), what is artifact? A) significant abnormality discovered by the paramedic B) The complete absence of electrical activity on the ECG C) Electrical interference that can make diagnosis difficult D) Improper lead placement, resulting in misdiagnosis

C Artifact refers to an ECG tracing that is the result of interference, such as patient movement, deep breathing, or muscle tremors, rather than the heart's electrical activity. Artifact can make diagnosis of the ECG difficult or impossible. To minimize artifact on the ECG, make sure the patient is lying supine; if the patient is experiencing respiratory distress, place him or her in a semi-Fowler's position. Ensure that the patient's arms are relaxed by his or her side and his or her feet are uncrossed.

A middle-aged man was found unresponsive by his wife. When you arrive at the scene, you assess the patient and determine that he is apneic and pulseless. You should: A) immediately begin CPR, reassess for a carotid pulse after 60 seconds, and then apply the AED. B) immediately apply the AED, analyze his cardiac rhythm, deliver a shock if indicated, and begin CPR. C) begin CPR starting with chest compressions, apply the AED as soon as possible, and request backup. D) perform CPR with a compression to ventilation ratio of 15:2, apply the AED, and request backup.

C As soon as you determine that a patient is unresponsive, pulseless, and apneic, you should begin CPR (starting with chest compressions), apply the AED as soon as possible, and deliver a shock (if indicated). The appropriate compression to ventilation ratio for adult CPR (one- or two-rescuer) is 30:2. A compression to ventilation ratio of 15:2 is used for two-rescuer infant and child CPR. Request a backup ambulance as soon as possible; however, do not interrupt CPR to do so. One EMT should perform CPR while the other radios for assistance. Continue CPR and reanalyze the patient's cardiac rhythm every 2 minutes. If indicated, deliver a single shock and immediately resume CPR, starting with chest compressions. If the AED gives a no shock message, resume CPR, starting with chest compressions. Continue CPR, rhythm analysis every 2 minutes, and defibrillation (if indicated), until backup arrives or the patient starts to move.

Aspirin may be contraindicated in patients with: A) glaucoma. B) diabetes. C) stomach ulcers. D) ibuprofen allergy.

C Aspirin (acetylsalicylic acid [ASA]) inhibits platelet aggregation, thus preventing clots from forming or preventing an existing clot from getting bigger. Aspirin, in a dose of 160 to 325 mg, should be administered to patients experiencing acute coronary syndrome (ie, unstable angina, acute myocardial infarction) as soon as possible. Aspirin is absolutely contraindicated for patients who are allergic to salicylates. Because aspirin prolongs bleeding time, it may be contraindicated for patients with stomach ulcers; therefore, you should contact medical control before giving aspirin to such patients. Aspirin is not contraindicated for patients with glaucoma or diabetes. Ibuprofen, the active ingredient in Motrin and Advil, is a nonsteroidal anti-inflammatory drug (NSAID), not a salycilate.

A 50-year-old man presents with crushing chest pain that suddently began about 30 minutes ago. He is diaphoretic and anxious. The EMT should: A) obtain baseline vital signs. B) apply supplemental oxygen. C) administer chewable aspirin. D) perform a complete physical exam.

C Aspirin (up to 325 mg) has clearly been shown to reduce mortality and morbidity from acute myocardial infarction (AMI) and should be given as soon as possible to patients with suspected cardiac chest pain (unless they are allergic to aspirin). Not all patients get oxygen, even those experiencing AMI. Give oxygen if the patient is hypoxemic (oxygen saturation less than 94%) or is experiencing respiratory distress. Clearly, it is important to perform a physical exam and obtain vital signs. Of the interventions listed, however, aspirin administration has the highest priority in this patient.

Which of the following is a common side effect of nitroglycerin? A) Nausea B) Anxiety C) Headache D) Hypertension

C Because nitroglycerin (NTG) causes vasodilation, including the vessels within the brain, cerebral blood flow increases following its administration. This often causes a pounding headache for the patient. As uncomfortable as it is for the patient, headaches are a common and expected side effect of the drug. The vasodilatory effects of nitroglycerin could result in hypotension; therefore, the patient's blood pressure should be carefully monitored. Nausea and anxiety are common symptoms of acute coronary syndrome; they are not common side effects of nitroglycerin.

The maximum pressure generated in the arms and legs during contraction of the left ventricle is called: A) afterload. B) pulse pressure. C) systolic blood pressure. D) diastolic blood pressure.

C Blood pressure is the force of circulating blood against the arterial walls. Systolic blood pressure (SBP) is the maximum pressure generated in the arms and legs during contraction of the left ventricle, during the time period known as systole. As the left ventricle relaxes in the stage known as diastole, the arterial pressure falls. When the left ventricle relaxes, the aortic valve closes and blood flow between the left ventricle and the aorta stops. The diastolic blood pressure (DBP) is the pressure exerted against the arterial walls while the left ventricle is at rest, during the time period known as diastole. Pulse pressure is the numeric difference between the SBP and DBP; it represents the force generated by the heart each time it contracts. If the patient's BP is 120/80 mm Hg, the pulse pressure would be 40 mm Hg. Afterload refers to the amount of resistance the left ventricle must contract against. Conditions such as hypertension cause an increase in afterload; as the arteries narrow, the left ventricle must work harder to overcome the increases resistance to forward blood flow.

Which of the following structures is the primary pacemaker, which sets the normal rate for the heart? A) Bundle of His B) Purkinje fibers C) Sinoatrial node D) Atrioventricular node

C Cardiac pacemakers are bundles of nerves that generate electrical impulses and conduct them to the cardiac cells, resulting in contraction of the myocardium (heart muscle). In a normal healthy heart, the sinoatrial (SA) node is the primary pacemaker that sets the inherent rate for the heart. The SA node generates electricity at a rate of 60 to 100 electrical discharges per minute; hence the normal adult heart rate is 60 to 100 beats/min. The atrioventricular (AV) node serves as the heart's secondary pacemaker; if the SA node fails, the AV node resumes the pacing function of the heart, although at a slower rate (40 to 60 beats/min). The bundle of His and Purkinjie fibers, located within the ventricles, may serve as tertiary pacemakers if the SA and AV nodes fail; their inherent pacing rate is 20 to 40 beats/min.

While transporting an elderly woman who was complaining of nausea, vomiting, and weakness, she suddenly becomes unresponsive. You should: A) analyze her cardiac rhythm with the AED. B) open her airway and ensure that it is clear. C) assess for signs of breathing and a pulse. D) place her on her side in case she vomits.

C If a patient is found unresponsive or becomes unresponsive in your presence, your first action should be to assess for breathing and a pulse; this assessment can occur simultaneously and should take no more than 10 seconds. If the patient is breathing adequately and has a pulse, position her on her side and administer oxygen if needed. If the patient has a pulse but is not breathing, open the airway and provide rescue breathing. If the patient is not breathing (or has agonal gasps) and does not have a pulse, begin CPR (starting with chest compressions) and apply the AED as soon as possible. If you are transporting a patient who becomes unresponsive, pulseless, and apneic, you should begin CPR and instruct your partner to stop the ambulance and prepare the AED.

You are treating a 60-year-old man in cardiac arrest. After delivering a shock with the AED and performing CPR for 2 minutes, you achieve return of spontaneous circulation. Your next action should be to: A) provide rapid transport to the hospital. B) reanalyze his rhythm for confirmation. C) assess his airway and ventilatory status. D) remove the AED and apply 100% oxygen.

C If return of spontaneous circulation (ROSC) occurs (eg, a palpable pulse is restored), your first action should be to reassess the patient's airway and ventilatory status. If the patient remains apneic, continue rescue breathing at a rate of 10 breaths/min (one breath every 6 seconds). If the patient is breathing adequately, administer supplemental oxygen in a concentration that is sufficient to maintain an oxygen saturation that is between 92% and 98%. After reassessing the airway and breathing, and treating the patient accordingly, you should prepare for immediate transport. Because of the high risk that cardiac arrest can recur following resuscitation, you should not remove the AED pads; simply turn the AED off instead. Analysis of the patient's cardiac rhythm is not indicated because he now has a pulse.

Which of the following structures stimulates the ventricles and causes them to contract? A) Sinus node B) Sinoatrial node C) Purkinje fibers D) Atrioventricular node

C In a normal heart, the sinoatrial (SA) node, also referred to as the sinus node, is the heart's primary pacemaker. Electrical impulses generated by the sinus node travel throughout both atria via special conduction pathways, causing simultaneous atrial contraction. The impulse continues down the conduction pathway to the atrioventricular (AV) node. At the AV node, the impulse is delayed for a brief period of time in order to allow the ventricles to fill with blood. The impulse then exits the AV node and spreads throughout both ventricles via the bundle of His, the right and left bundle branches, and the Purkinje fibers, causing the ventricular muscle cells to contract.

After applying the AED to an adult patient in cardiac arrest, you analyze her cardiac rhythm and receive a shock advised message. Emergency medical responders, who arrived at the scene before you, tell you that bystander CPR was not in progress upon their arrival. You should: A) perform CPR for 2 minutes and then defibrillate. B) detach the AED and prepare for immediate transport. C) deliver the shock as indicated followed immediately by CPR. D) notify medical control and request permission to cease resuscitation.

C Information obtained at the scene—whether from emergency medical responders or family members—is not always accurate. If the AED advises you to defibrillate, you should do so and then immediately begin or resume CPR, starting with chest compressions. During CPR, ensure adequate oxygenation and ventilation, minimize interruptions in chest compressions, obtain the patient's medical history if possible, and contact medical control as needed. At no time during resuscitative efforts should you detach the AED from the patient. Reanalyze the patient's cardiac rhythm after 2 minutes of CPR and follow the voice prompts.

The chest pain associated with an acute coronary syndrome is often described as: A) sharp. B) stabbing. C) pressure. D) cramping.

C The pain associated with acute coronary syndrome (ie, unstable angina, acute myocardial infarction) is commonly described as a feeling of pressure or heaviness, discomfort, or a crushing pain. Stabbing or sharp (pleuritic) pain is often associated with conditions such as a spontaneous pneumothorax or pulmonary embolism. Do not rule out a cardiac problem just because the patient does not have the "classic" quality of pain, especially if the patient has a history of cardiac problems.

When the vital organs of an 85-year-old patient need additional blood flow, the heart may not be able to meet the increased need because: A) stroke volume increases as a person ages. B) blood pressure decreases as a person ages. C) the reserve capacity of the heart is reduced as a person ages. D) the blood vessels become more elastic as a person ages.

C Like other body systems, the cardiovascular system undergoes changes as we get older. As the heart's muscle mass and tone decrease, the amount of blood pumped out of the heart per beat (stroke volume) is reduced. The residual (reserve) capacity of the heart is also reduced; therefore, when the vital organs of the body need additional blood flow, the heart is less able to meet that need as effectively as a younger person's heart. The vascular system also is affected by the aging process. Arteriosclerosis (hardening of the arteries) can develop, affecting perfusion of the tissues. In arteriosclerosis, the arteries become less elastic and more narrowed; as a result, blood pressure typically increases with age.

Which of the following would clearly be detrimental to a patient in cardiac arrest? A) Ventilating just until the chest rises B) Performing CPR before defibrillation C) Interrupting CPR for more than 10 seconds D) Ventilating without supplemental oxygen

C Major emphasis is placed on minimizing interruptions in CPR. Even brief interruptions cause a drop in coronary and cerebral perfusion. If you must interrupt CPR, do not exceed 10 seconds. Every effort should be made to maintain a high chest compression fraction (CCF). CCF is the percentage of time during a resuscitation in which chest compressions are being performed. It is preferable to ventilate a patient with supplemental oxygen, but failing to do so will not be nearly as detrimental as interrupting CPR for extended periods of time. You should ventilate the patient just until the chest visibly rises; ventilations that are too forceful or too fast can cause hyperinflation of the lungs, which may reduce blood return to the heart. When caring for any patient in cardiac arrest, you should immediately begin CPR, and then apply the AED as soon as possible.

Following administration of nitroglycerin to a man with crushing chest pressure, he experiences a significant increase in his heart rate. This is MOST likely the result of: A) a cardiac dysrhythmia. B) preexisting hypertension. C) a drop in blood pressure. D) coronary vasoconstriction.

C Nitroglycerin (NTG) is a vasodilator drug. It dilates not only the coronary arteries, but also other arteries in the body. In some patients, NTG may cause a drop in blood pressure, especially if they are taking other medications that cause vasodilation (eg, erectile dysfunction drugs [Viagra, Levitra, Cialis]). In response to a drop in blood pressure, the nervous system attempts to compensate by increasing the heart rate (tachycardia). Common side effects of NTG include a headache, a burning sensation under the tongue, and a bitter taste in the mouth. Always assess the patient's vital signs both before and after administering nitroglycerin.

Which of the following patients would MOST likely present with vague or unusual symptoms of an acute myocardial infarction? A) 55-year-old obese female B) 66-year-old male with angina C) 72-year-old female with diabetes D) 75-year-old male with hypertension

C Not all patients experiencing acute myocardial infarction (AMI) present with the classic signs and symptoms one would expect. Middle-aged men often minimize their symptoms and attribute their chest pain or discomfort to indigestion. Some patients, however, do not experience any pain. In particular, elderly women with diabetes may present with vague, unusual, or atypical symptoms of AMI; their only presenting complaint may be fatigue or syncope. Do not rule out a cardiac problem just because a patient is not experiencing chest pain, pressure, or discomfort; this is especially true in elderly females with diabetes.

Which of the following assessment findings is LEAST suggestive of a cardiac problem? A) Rapid, irregular heart rate B) Anxiety and pale, cool skin C) Palpable pain to the chest D) Nausea and epigastric pain

C Signs and symptoms of cardiac compromise include nonreproducible pain, pressure, or discomfort in the chest or epigastric region; nausea; pale, cool, clammy (diaphoretic) skin; and an irregular pulse that is either fast or slow. Pain of cardiac origin typically is not reproducible by palpation. Palpable pain to the chest suggests a musculoskeletal problem, not a cardiac problem. However, because some patients with a cardiac problem present atypically, you should transport any patient with chest pain, pressure, or discomfort to the hospital.

Use of the automated external defibrillator is contraindicated in patients who: A) are between 1 and 8 years of age. B) experienced a witnessed cardiac arrest. C) are apneic and have a weak carotid pulse. D) have a nitroglycerin patch applied to the skin.

C The AED is applied only to patients in cardiac arrest (eg, pulseless and apneic), whether the arrest was witnessed or unwitnessed. According to the 2015 guidelines for CPR and Emergency Cardiac Care (ECC), AEDs can safely be used in infants and children less than 8 years of age in conjunction with a dose-attenuating system (energy reducer) and pediatric pads. However, if pediatric pads and an energy reducer are unavailable, adult AED pads should be used. A nitroglycerin patch is not a contraindication to the use of an AED; simply remove the patch (with gloved hands) and apply the AED as usual.

The middle, muscular layer of the heart is called the: A) epicardium. B) pericardium. C) myocardium. D) endocardium.

C The heart has four layers. The inner layer is called the endocardium, the middle layer is composed of muscle and is called the myocardium (myo = muscle), and the outer layer of the heart itself is called the epicardium. The pericardium, which is a thin, fibrous membrane, encapsulates the entire heart.

Sudden cardiac arrest in the adult population MOST often is the result of: A) myocardial infarction. B) respiratory failure. C) a cardiac dysrhythmia. D) accidental electrocution.

C The most common cause of sudden cardiac arrest (SCA) in the adult population is a cardiac dysrhythmia--usually ventricular fibrillation. This fact underscores the importance of early defibrillation to shock the heart back into a perfusing rhythm. Evidence has shown that SCA--again, most often the result of an arrhythmia--occurs in up to 40% of patients experiencing an acute myocardial infarction (AMI). The risk of SCA is highest within the first few hours following the onset of an AMI. Respiratory failure is the most common cause of cardiac arrest in children, not adults. Children generally have healthy hearts and rarely experience cardiac arrest due to a primary cardiac event.

A 49-year-old woman presents with a headache, nausea, and ringing in her ears. She is conscious and alert and states that she has hypertension and type 2 diabetes. Her BP is 202/114 mm Hg, her pulse is 60 beats/min, and her respirations are 16 breaths/min. What should you do? A) Place her in a supine position B) Give her one tube of oral glucose C) Prepare for immediate transport D) Summon an ALS unit to the scene

C The patient's presentation is indicative of acute hypertensive crisis. Her blood pressure is severely elevated and she is experiencing other symptoms one might expect with such a high blood pressure (ie, headache, nausea, ringing in the ears). You should place her in a position of comfort, which is usually a semisitting position; lying her supine would likely only make her headache worse. Prompt transport to the hospital is essential so her blood pressure can be lowered in a controlled setting. Without knowing her blood glucose level, proceeding with oral glucose would not be appropriate. If it does not delay transport, you could consider summoning an ALS ambulance to the scene. It is far more important, however, to get her to a definitive care facility

Freshly oxygenated blood returns to the heart via the: A) aorta. B) vena cavae. C) pulmonary veins. D) pulmonary arteries.

C The pulmonary veins are the only veins that carries oxygen-rich blood. They carry blood from the lungs back to the left atrium. All other veins in the human body, including the venae cavae, carry deoxygenated blood back to the heart. The aorta is the largest artery in the body and branches immediately from the left ventricle, carrying freshly oxygenated blood to the rest of the body. The pulmonary arteries carry deoxygenated blood from the right ventricle to the lungs for reoxygenation.

A 66-year-old female presents with an acute onset of confusion, slurred speech, and an inability to move her right arm and leg. Her airway is patent and she is breathing adequately. Which of the following is the MOST important initial information to obtain about this patient? A) Her initial blood pressure reading B) What she was doing when this began C) When she was last seen normal D) Whether or not her pupils are equal

C This patient is experiencing signs of an acute ischemic stroke. She may be a candidate for fibrinolytic therapy (drugs that dissolve blood clots) if her symptoms are of less than 3 hours' duration. Some patients may be candidates for fibrinolytic therapy for up to 4.5 hours after the onset of symptoms. It is vital to determine exactly (or as close to as possible) when the patient was last seen normal, and pass this information along to the receiving facility. The treatment options for stroke are severely limited if more than 3 hours have past since the symptoms began. Even if 3 hours have passed, prompt action on your part is essential. Assessment of the patient's blood pressure, pupils, and events that preceded the symptoms is important; however, identifying the patient as a candidate for an intervention that may reverse the stroke is critical and will afford her the greatest chance for a positive outcome.

A rapid heart rhythm, usually at a rate of 150 to 200 beats/min, that originates in the ventricles and can cause hemodynamic compromise is called: A) asystole. B) ventricular fibrillation. C) ventricular tachycardia. D) pulseless electrical activity.

C Ventricular tachycardia (V-Tach) is a rapid heart rhythm, usually at a rate of 150 to 200 beats/min, that originates in the ventricle instead of the atrium. V-Tach usually does not allow adequate time between beats for the left ventricle to fill with blood; therefore, the blood pressure may fall. V-Tach may occur with or without a pulse. Ventricular fibrillation (V-Fib) is a disorganized, chaotic dysrhythmia that does not produce a pulse; it occurs when cardiac cells randomly discharge to the point that regular cardiac contraction is impossible. Asystole (flatline) represents an absence of cardiac electrical and mechanical activity; obviously, asystole does not produce a pulse. Pulseless electrical activity (PEA) is a phenomenon in which organized electrical activity is present on the cardiac monitor, despite the absence of a pulse

Chest compression effectiveness is MOST effectively assessed by: A) listening for a heartbeat with each compression. B) carefully measuring the depth of each compression. C) palpating for a carotid pulse with each compression. D) measuring the systolic blood pressure during compressions.

C When chest compressions are in progress, the most reliable method of determining their effectiveness is to palpate for a carotid or femoral pulse. If compressions are of adequate depth for the patient's age, you should be able to feel a pulsation during each compression. It should be noted that you may not be able to feel a pulse in some patients, despite adequately performed chest compressions. For example, if the patient has lost a significant amount of blood, there is less volume to create the pressure wave against the arterial wall. If the patient has a pericardial tamponade or tension pneumothorax (examples of obstructive shock), then the forward flow of blood may be obstructed; in these cases, you may not feel a pulse during chest compressions

A 56-year-old man is found to be pulseless and apneic. His wife states that he collapsed about 5 minutes ago. As your partner gets the AED from the ambulance, you should: A) provide rescue breaths until the AED is ready. B) open the airway and give 2 rescue breaths. C) begin CPR, starting with chest compressions. D) ask the wife if the patient has a living will.

C When you arrive on scene and determine that a patient is in cardiac arrest, you should immediately begin CPR, starting with chest compressions. Perform 30 chest compressions and then open the airway and deliver 2 rescue breaths. Chest compressions are a crucial part of cardiopulmonary resuscitation and must be started without delay. Apply the AED as soon as it is available. In the interest of this patient, whose arrest interval is short, you should begin resuscitative efforts immediately. In some cases, it is appropriate to inquire about the presence of a living will; however, this should be done after resuscitative efforts have begun.

Which of the following is an expected finding in patients who have a left ventricular assist device (LVAD) with a continuous flow pump? A) cyanotic skin. B) hypertension. C) peripheral edema. D) absence of pulses.

D A left ventricular assist device (LVAD) is used to enhance left ventricular pumping. LVADs are commonly implanted in patients with severe heart failure, those who are awaiting a heart transplant, or those who need a temporary boost following acute myocardial infarction. Several types of LVADs exist; the most common ones have an internal pump unit and an external battery pack. These pumps may be pulsatile, meaning they pump in pulsations just like the natural heart, or they may be continuous, in which case the patient will not have any palpable pulses. If pulses are not palpable, then a blood pressure will not be obtainable. You would not expect to encounter cyanosis in any patient with an LVAD, whether the pump is pulsatile or continuous; if you do, an oxygenation problem exists. Peripheral edema is also not expected in patients with an LVAD.

Which of the following statements regarding the automated external defibrillator (AED) is correct? A) The AED should be applied to patients at risk for cardiac arrest. B) AEDs will analyze the patient's rhythm while CPR is in progress. C) The AED should not be used in patients with an implanted defibrillator. D) AEDs can safely be used in infants and children less than 8 years of age.

D According to the 2020 guidelines for CPR and Emergency Cardiac Care (ECC), the AED can safely be used in infants and children less than 8 years of age. Although a manual defibrillator is preferred in infants, an AED can be used. When using the AED in infants and children, you should use pediatric pads and a dose-attenuating system (energy reducer); however, if these are not available, adult AED pads should be used. The AED should be applied only to patients in cardiac arrest; if a patient is at risk for cardiac arrest, have the AED ready but not applied. The AED will not analyze the cardiac rhythm if the patient is moving (ie, CPR is in progress). AEDs can be used in patients with an automated implanted cardioverter/defibrillator (AICD) or implanted pacemaker; ensure that the pads are at least 1 inch away from the implanted device.

While assessing a patient with chest pain, you note that his pulse is irregular. This indicates: A) acute myocardial infarction or angina pectoris. B) a dysfunction in the left side of the patient's heart C) high blood pressure that is increasing cardiac workload. D) abnormalities in the heart's electrical conduction system.

D An irregular pulse indicates abnormalities in the electrical conduction system of the heart. The electrical conduction system, beginning with the sinoatrial node as the primary pacemaker, is responsible for initiating the electrical impulses that stimulate the myocardium to contract. An irregular pulse could indicate a potentially lethal dysrhythmia that could result in cardiac arrest. You should document an irregular pulse and report this important finding to the emergency department.

A 62-year-old male with a history of coronary artery disease began experiencing chest pain and nausea while on his daily jog. After sitting down and resting for 5 minutes, his pain and nausea resolve. Which of the following BEST describes this patient's condition? A) Musculoskeletal chest pain B) Acute myocardial infarction C) Unstable angina pectoris D) Stable angina pectoris

D Angina pectoris is the principle symptom of coronary artery disease (CAD). It occurs when the heart's demand for oxygen exceeds its supply, usually during periods of physical or emotional stress when the heart is working hard. When the increased oxygen demand goes away, the pain typically resolves. Some patients with angina may also experience shortness of breath, nausea, or diaphoresis. Angina is classified as being "stable" or "unstable." Stable angina is characterized by cardiac-related chest pain that resolves with rest and/or nitroglycerin. Unstable angina is characterized by cardiac-related chest pain that occurs in response to progressively less physical exertion (for example, during rest). Unstable angina is also referred to as pre-infarction angina because it indicates a more severe degree of CAD and can lead to acute myocardial infarction if untreated. In contrast to stable angina, unstable angina typically does not resolve with rest and/or nitroglycerin. Given the patient's medical history, his chest pain should not be assumed to have simply been of a musculoskeletal origin.

How can you help maximize cardiac output during CPR? A) Compress the chest at a rate of no more than 100/min B) Ventilate the patient through an advanced airway device C) Deliver rescue breaths until the chest expands widely D) Allow the chest to fully recoil in between compressions

D Cardiac output is the amount of blood ejected from the left ventricle per minute. Bearing in mind that even the best-performed CPR produces only a fraction of what the patient's cardiac output would otherwise be, there are several actions that you must take to help maximize this. Allowing the chest to fully recoil in between compressions will help draw blood back to the heart; if more blood returns to the heart, more blood can be pumped from the heart with chest compressions. Do not lean on the patient's chest between compressions. Delivering each rescue breath over a period of 1 second, just enough to produce visible chest rise, will also help maximize cardiac output. If ventilations are given too fast or too forcefully, intrathoracic pressure will increase, resulting in a decrease in the amount of blood that returns to the heart; as a result, cardiac output will decrease. Ventilations are delivered no differently if an advanced airway device (ie, ET tube, multilumen airway, supraglottic airway) has been inserted. Deliver chest compressions at a rate of 100 to 120 per minute to a depth of at least 2 inches (at least one-third the depth of the chest in infants and children). Chest compression depth in the adult should not exceed 2.4 inches; however, this is not possible to determine without a CPR device that provides immediate feedback

Tachycardia can be detrimental to a patient who is experiencing a cardiac problem because it causes: A) increased cardiac filling in between beats. B) a profound decrease in oxygen consumption. C) an associated increase in breathing difficulty. D) increased cardiac oxygen usage and demand.

D Many patients experiencing a cardiac problem are tachycardic (heart rate greater than 100 beats/min); others are bradycardic (heart rate less than 60 beats/min). As the heart beats faster, it consumes, and therefore requires, more oxygen. This can be detrimental to the patient because the heart is already deprived of oxygen (ischemia) and may not be able to accommodate the increased need for oxygen; this may worsen the ischemia, potentially causing further cardiac damage. A very fast heart rate (greater than 150 beats/min) may cause hypotension due to a decrease in cardiac output; it occurs because the ventricles are not adequately filling with blood in between beats. Some patients with a cardiac problem may have difficulty breathing, which may be the result of congestive heart failure and pulmonary edema; it is not caused by the tachycardia itself.

In which of the following situations would nitroglycerin MOST likely be administered? A) Recent use of Cialis B) Systolic BP of 90 mm Hg C) The presence of a head injury D) History of cardiac bypass surgery

D Nitroglycerin (NTG) is a vasodilator drug used to relieve chest pain in patients with cardiac compromise by dilating the coronary arteries and improving blood flow to the heart. Because of its vasodilator effects, it should not be given to patients who have a systolic BP less than 100 mm Hg or to patients who have recently (within the past 24 to 48 hours) taken erectile dysfunction (ED) drugs (eg, sildenafil [Viagra], vardenafil [Levitra], tadalafil [Cialis]). ED drugs also cause vasodilation and may cause significant hypotension if given together with NTG. You should also avoid NTG in patients with a head injury; dilation of the cerebral blood vessels may worsen intracranial pressure caused by the head injury. By itself, a history of cardiac bypass surgery does not contraindicate the use of NTG.

After administering nitroglycerin to a patient with chest discomfort, it is MOST important for you to: A) ask the patient if the discomfort has improved. B) find out how long the discomfort has been present. C) position the patient supine and transport immediately. D) reassess the patient's blood pressure within 5 minutes.

D Nitroglycerin (NTG) relaxes the muscle of blood vessel walls, dilates the coronary arteries, increases blood flow and the supply of oxygen to the heart muscle (myocardium), and decreases the workload of the heart. NTG also dilates blood vessels in other parts of the body, potentially resulting in hypotension. For this reason, you should reassess the patient's blood pressure within 5 minutes after each dose of NTG. If the systolic blood pressure is less than 100 mm Hg, do not give anymore NTG. If significant hypotension occurs, position the patient supine and transport without delay. Asking the patient if his or her chest pain or discomfort has improved following NTG helps you determine if the drug is working and whether additional dosing is needed; however, detecting hypotension is clearly more important. You should determine when the chest pain or discomfort began during the focused history, which is typically performed before assisting a patient with his or her prescribed NTG.

Which of the following describes pulseless electrical activity (PEA)? A) disorganized, chaotic quivering of the heart muscle that does not generate a pulse B) The presence of a palpable pulse in the absence of any electrical activity in the heart C) A rapid cardiac rhythm that does not produce a pulse, but responds to defibrillation D) Any organized cardiac rhythm, slow or fast, that does not produce a palpable pulse

D Pulseless electrical activity (PEA) is a condition in which the heart produces organized electrical activity (slow or fast), despite the absence of a palpable pulse. A disorganized, chaotic quivering of the heart muscle that does not generate a pulse is called ventricular fibrillation (V-Fib), and is treated with defibrillation. Defibrillation is not indicated for patients with PEA; it is indicated only for patients with V-Fib or pulseless ventricular tachycardia (V-Tach). If the AED gives a no shock message and the patient is still pulseless, he or she is either in asystole or PEA, neither of which is a shockable rhythm

The MOST important initial treatment for a patient whose cardiac arrest was witnessed is: A) defibrillation. B) cardiac drug therapy. C) rapid transport. D) high-quality CPR.

D Regardless of whether a patient's cardiac arrest is witnessed or unwitnessed, the single most important initial treatment is high-quality CPR. Delays in performing CPR have been clearly linked to poor patient outcomes. After CPR has been initiated, apply the AED as soon as it is available. Cardiac drug therapy may enhance the patient's chance of survival. There is evidence suggesting that transport to the hospital with CPR in progress does not improve survival; resuscitation at the scene may be more beneficial for the patient; follow your local protocols and current research trends. Minimally interrupted, high-quality CPR, however, is clearly linked to good patient outcomes.

A 67-year-old woman presents with acute weakness and nausea. She is conscious and alert and her skin is cool and clammy. Her BP is 140/88 mm Hg, pulse is 70 beats/min and regular, and respirations are 16 breaths/min and unlabored. Her medical history includes hypertension and diabetes, and she advises that she has an implanted pacemaker. What should you suspect? A) Pacemaker malfunction B) Acute hypertensive crisis C) Infection with influenza D) Acute myocardial infarction

D Remember this: elderly females with diabetes are the most likely to present with vague, unusual, or nonclassic symptoms of acute myocardial infarction. Acute weakness, nausea, and diaphoresis in this patient population should make you very suspicious for a cardiac event. Pacemaker malfunction is not causing her symptoms; if this was the cause, she would be bradycardic. Her blood pressure is somewhat elevated, but not so high as to cause acute hypertensive crisis. In addition, she is not experiencing any symptoms of acute hypertensive crisis, such as headache, ringing in the ears, or photophobia. Infection with the influenza virus (the flu) is unlikely; the flu does not present acutely, plus you would expect the patient's skin to be warm or hot from fever, not cool.

When assessing a patient who complains of chest pain, which of the following questions would you ask to assess the "R" in OPQRST? A) Did the pain begin suddenly or gradually? B) What were you doing when the pain began? C) Is there anything that makes the pain worse? D) Is the pain in one place or does it move around?

D The "R" in OPQRST stands for radiation or referred pain. An appropriate way to determine whether the pain radiates is to ask the patient if the pain remains in one place or if it moves around. When determining if the patient has referred pain, ask the patient if he or she hurts somewhere other than his or her chest. If you use the term "radiating pain," chances are the patient will not understand what you are asking.

When an electrical impulse reaches the atrioventricular node, it is briefly delayed so that: A) the aortic valve can open. B) both of the atria can relax. C) the ventricles can contract. D) ventricular filling can occur.

D The heart functions as a double pump; when the atria are contracting, the ventricles are filling, and when the ventricles are contracting, the atria are filling. In order to maintain the double pump effect, the electrical impulse coming from the sinoatrial node must undergo a brief delay at the atrioventricular node; this delay allows the ventricles to fill with blood. Most of the blood that fills the ventricles does so passively, with final filling occurring during atrial contraction; this is called the atrial kick. The brief delay at the AV node does not affect when the aortic valve opens, when the ventricles contract, or when the atria relax.

Blood is returned from the kidneys to the heart by way of the: A) renal arteries. B) splenic arteries. C) coronary sinus. D) inferior vena cava.

D The inferior vena cava returns deoxygenated blood from the abdomen, kidneys, and legs back to the right side of the heart. The renal arteries branch from the aorta and supply oxygenated blood to the kidneys. The splenic arteries also branch from the aorta and supply oxygenated blood to the spleen. The coronary sinus returns deoxygenated blood from the heart to the right atrium

A 71-year-old woman presents with an acute onset of confusion, slurred speech, and left-sided weakness. She is conscious and her airway is patent. Her BP is 180/94 mm Hg, her pulse is 70 beats/min and irregular, her respirations are 14 breaths/min and unlabored, and her oxygen saturation is 97% on room air. What should you do? A) Administer up to 325 mg of chewable aspirin B) Position her supine and assist her ventilations C) Give oxygen via nasal cannula at 4 to 6 L/min D) Keep her sitting up and transport without delay

D The patient's presentation is obvious for a stroke. Second ONLY to addressing problems with airway, breathing, and circulation, the most important intervention is to transport the patient to a stroke center, where fibrinolytic medications can be given or other interventions can be performed. Keep the patient sitting up in order to protect her airway, and transport without delay. The patient in this scenario is breathing adequately; therefore, assisted ventilation is not indicated. Although her BP is elevated (common with a stroke), this is not treated by the EMT in the field. The patient's oxygen saturation does not indicate hypoxemia and she is not experiencing respiratory distress; therefore, supplemental oxygen is not indicated. Do NOT administer aspirin to a patient with stroke-like symptoms; if the cause of the stroke is hemorrhage in the brain, aspirin could worsen the situation

Prior to being reoxygenated in the lungs, blood passes through the: A) mitral and tricuspid valves. B) aortic and tricuspid valves. C) aortic and pulmonic valves. D) tricuspid and pulmonic valves.

D There are two valves that blood must pass through before being reoxygenated in the lungs, the tricuspid and the pulmonic. After returning to the body, blood passes from the right atrium, through the tricuspid valve, to the right ventricle. It then passes through the pulmonic valve, into the pulmonary arteries, and to the lungs. The mitral valve is located between the left atrium and left ventricle; blood passes through this valve after being reoxygenated in the lungs. Oxygenated blood moves through the aortic valve, into the aorta, and out to the body.

Treatment for a patient with congestive heart failure and shortness of breath may include: A) prophylactic suctioning of the airway. B) hyperventilation with a bag-valve-mask. C) supine positioning and elevation of the legs. D) up to three doses of sublingual nitroglycerin.

D Treatment for patients with congestive heart failure (CHF) includes supplemental oxygen as needed to maintain an oxygen saturation equal to or greater than 94%, continuous positive airway pressure (CPAP), ventilatory assistance with a bag-mask device if needed (do NOT hyperventilate the patient), placing the patient in an upright or sitting position to facilitate breathing (a supine position will clearly make it more difficult to breathe), monitoring the patient's vital signs, and transporting to the hospital without delay. Suction the airway only if there are secretions in the mouth; prophylactic suctioning is not indicated. Nitroglycerin (NTG) may be of value if the patient is not hypotensive and he or she has the medication prescribed to him or her. As a vasodilator, NTG causes systemic venous pooling of blood, which reduces the amount of blood returned to the heart (preload) and, therefore, the amount of blood available to back up in the lungs. Follow your local protocols or contact medical control as needed regarding the use of NTG for patients with CHF.

Which of the following statements regarding one-rescuer CPR is correct? A) You should assess the patient for a pulse after 3 cycles of CPR. B) A compression to ventilation ratio of 15:2 should be delivered. C) Ventilations should be delivered over a period of 2 to 3 seconds. D) The chest should be allowed to fully recoil after each compression

D When performing CPR on any patient, you should allow the chest to fully recoil after each compression; do not lean on the chest between compressions. Incomplete chest recoil causes increased intrathoracic pressure, which may impair blood return to the heart. Assess the patient's pulse after every 5 cycles (about 2 minutes) of CPR; take no longer than 5 to 10 seconds to do this. A compression to ventilation ratio of 30:2 should be used during all adult and one-rescuer CPR (adult, child, and infant), except for newborns. A compression to ventilation ratio of 15:2 is used during two-rescuer infant and child CPR. Ventilations should be delivered over a period of 1 second each, just enough to produce visible chest rise.

A middle-aged female with a history of hypertension and high cholesterol complains of chest discomfort. She asks you to take her to the hospital where her personal physician practices, which is 15 miles away. Her blood pressure is 130/70 mm Hg, pulse is 84 beats/min and regular, and respirations are 18 breaths/min and unlabored. Which of the following actions is clearly NOT appropriate for this patient? A) Giving oxygen via nasal cannula B) Taking her to her choice hospital C) Contacting her physician via phone D) Allowing her to walk to the ambulance

D You should NEVER allow a patient with a possible cardiac problem to walk to the ambulance. This causes exertion, which increases cardiac oxygen consumption and demand and could worsen his or her condition. Give the patient oxygen in a concentration sufficient to maintain his or her oxygen saturation equal to or greater than 94%. In general, you should transport patients to the hospital of their choice. However, transport to a closer hospital should be considered if you believe the patient is unstable or is at high risk for becoming unstable. If necessary, consult with the patient's physician via phone to determine if he or she thinks the patient should go to a closer hospital

Your assessment of a middle-aged female with chest pressure reveals that she is confused, is diaphoretic, and has a blood pressure of 70/50 mm Hg. In caring for this patient, it is MOST important for you to: A) request an ALS unit to respond to the scene. B) assist her with her nitroglycerin if she has any. C) reassess her vital signs at least every 5 minutes. D) prepare for immediate transport to the hospital.

D Your patient is in shock, which is most likely the result of heart failure (cardiogenic shock). Immediate transport to the closest appropriate hospital is critical. Assess for and manage problems with airway, breathing, and circulation, and then get on the road! She needs definitive care that can be provided only at the hospital. Reassess her vital signs at least every 5 minutes, but do it en route! If you can rendezvous with an ALS unit en route, do so. However, you should not remain at the scene to wait for them. Nitroglycerin is clearly contraindicated for this patient; her BP is dangerously low.

Which of the following clinical presentations is MOST consistent with right heart failure? A) Tachycardia, poor skin turgor, and hypotension B) Orthopnea, crackles in the lungs, and tachycardia C) Peripheral edema and jugular venous distention D) Paroxysmal nocturnal dyspnea and hypertension

When the right side of the heart fails, blood backs up beyond the right atrium and into the systemic circulation. Right heart failure produces signs such as peripheral edema, abdominal distention and pain, and jugular venous distention. When the left side of the heart fails, blood backs up beyond the left atrium and into the lungs. Left heart failure produces signs such as dyspnea on exertion, positional dyspnea (orthopnea), paroxysmal nocturnal dyspnea, coughing up blood (hemoptysis), and crackles in the lungs. In addition, patients with left heart failure are commonly hypertensive and tachycardic. Tachycardia, poor skin turgor, and hypotension are clinical signs of severe dehydration, not heart failure.


Kaugnay na mga set ng pag-aaral

Taxes, Retirement, and other Insurance Concepts QUIZ

View Set

Physics and Math 1.2 Vectors and Scalars

View Set

Chapter 11: Finances & Career Planning Vocabulary

View Set

Biology: Quiz 3: BODY CONTROL AND INTERACTION WITH ENVIRONMENT

View Set

Chapter 15: Obsessive-Compulsive Related Disorders

View Set